Đến nội dung

poset

poset

Đăng ký: 14-04-2021
Offline Đăng nhập: Riêng tư
****-

#743510 Chứng minh rằng dãy $(\lfloor x\rfloor)_{n\in\m...

Gửi bởi poset trong 13-02-2024 - 15:02

Giả sử tồn tại số hai số nguyên dương $N$ và $k$ thỏa mãn $\lfloor x_{n+k}\rfloor=\lfloor x_n\rfloor$ với mọi số nguyên $n\ge N_1=N+1$. Vì $\lim_{n\to\infty}\frac{1}{q^n}=0$ nên tồn tại số nguyên dương $\alpha$ sao cho

\[|x_{N_1+k}-x_{N_1}|>\frac{1}{q^\alpha}.\tag{$\ast$}\]

Tiếp theo ta sẽ chỉ ra mâu thuẫn, bắt đầu với $\lfloor x_{N_1+\alpha+k}\rfloor=\lfloor x_{N_1+\alpha}\rfloor$ ta có

\[|x_{N_1+\alpha+k}-x_{N_1+\alpha}|<1\implies \Big|q\{x_{N_1+\alpha-1+k}\}-q\{x_{N_1+\alpha-1}\}\Big|<1.\]

Mặt khác $\lfloor x_{N_1+\alpha-1+k}\rfloor=\lfloor x_{N_1+\alpha-1}\rfloor$ dẫn đến

\[|x_{N_1+\alpha-1+k}-x_{N_1+\alpha-1}|=\Big|\{x_{N_1+\alpha-1+k}\}-\{x_{N_1+\alpha-1}\}\Big|<\frac{1}{q}.\]

Tiếp tục như thế ta có $|x_{N_1+k}-x_{N_1}|<\frac{1}{q^\alpha}$, mâu thuẫn với $(\ast)$.

 

P.s. Đọc lại thì thấy xét thiếu trường hợp  :mellow:

Nếu $x_{N_1}=x_{N_1+k}$ thì sao? Khi đó sẽ không tìm được $\alpha$.
Dữ kiện $x_1=q$ hay $q$ hữu tỉ không nguyên chưa được dùng, mặc dù nó không thừa. Ví dụ lấy $q=\frac{5}{2},x_1=\frac{5}{3}$ thì $x_n$ luôn bằng $\frac{5}{3}$, hay lấy $q$ là nghiệm lớn hơn $1$ của phương trình $x^3-x^2-2x+1=0$ thì dãy sẽ tuần hoàn với chu kì $2$.




#742926 Có bao nhiêu cách sắp xếp $2n$ quả bóng trên một đường thẳng

Gửi bởi poset trong 07-01-2024 - 18:13

Có $2n$ quả bóng được đánh số $1,2,...,n$ và tô $2$ màu đen trắng, không có $2$ quả nào vừa cùng màu vừa cùng số. Có bao nhiêu cách sắp xếp chúng trên một đường thẳng thỏa mãn cả hai điều kiện:
a) Quả bóng đen $a$ đứng trước quả bóng đen $b$ khi và chỉ khi quả bóng trắng $b$ đứng trước quả bóng trắng $a$.
b) Quả bóng đen $a$ đứng trước quả bóng trắng $b$ khi và chỉ khi quả bóng đen $b$ đứng trước quả bóng trắng $a$.




#742387 Có tồn tại hàm số khả vi cấp 1 trên khoảng (a,b) có đạo hàm không liên tục tạ...

Gửi bởi poset trong 07-12-2023 - 23:26

Có tồn tại không một hàm số khả vi cấp 1 tại mọi điểm thuộc khoảng (a,b) nhưng đạo hàm của nó không liên tục tại một điểm nào đó thuộc khoảng (a,b) ?  Nếu không hãy chứng minh rằng không tồn tại còn nếu có hãy cho ví dụ.

https://en.wikipedia...tiable_(not_C1)




#741874 $S_n = \sum_{i=1}^{n}|{1-\frac{u...

Gửi bởi poset trong 27-10-2023 - 23:25

Cho dãy $(u_n)$ thỏa $lim$$u_n=0$. Chứng minh rằng: $S_n=\sum_{i=1}^{n}\left | 1-\frac{u_{i+1}}{u_i} \right |$ phân kì.

Thêm điều kiện $u_n\neq 0,\forall n\in\mathbb{N}$ nữa. Đặt $c=1-\frac{1}{e}$, ta có $0<c<1$.

Bổ đề
$x\geq -c.ln(1-x),\forall x\in[0,c]$.

Chứng minh
Xét hàm $f(x)=x+c.ln(1-x)$ trên $[0,c]$. Ta có $f''(x)=\frac{c}{(1-x)^2}>0,\forall x\in[0,c]$, vậy $f$ là hàm lồi trên $[0,c]$ nên nhận giá trị nhỏ nhất tại biên, tại $x=0,x=c$ ta có $f=0$ nên $f(x)\geq 0\Rightarrow x\geq -c.ln(1-x),\forall x\in[0,c]$.

Đặt $v_n=1-\frac{u_{n+1}}{u_n}$ thì $u_{n+1}=u_m\prod_{i=m}^n(1-v_i)$ và $S_n=\sum_{i=1}^n|v_n|$. Giả sử $S_n$ hội tụ thì $\lim|v_n|=0$, từ đó chọn được $N$ sao cho $\forall n\geq N, |v_n|\leq c<1$, khi đó $1-v_n>0,\forall n\geq N$, từ đó bằng quy nạp ta dễ thấy $u_n$ cùng dấu với $u_N$ với mọi $n\geq N$. Theo Theorem ta có:
$S_n=\sum_{i=1}^n|v_n|=S_{N-1}+\sum_{i=N}^n|v_n|\geq S_{N-1}-c\sum_{i=N}^{n}ln(1-|v_n|)\geq S_{N-1}-c\sum_{i=N}^{n}ln(1-v_n)=S_{N-1}-c.ln\prod_{i=N}^{n}(1-v_n)=S_{N-1}-c.ln\frac{u_{n+1}}{u_N},\forall n\geq N$
Vì $u_n$ cùng dấu với $u_N$, $\lim u_n=0$ và $u_n\neq 0$ với $n\geq N$ nên:
$\frac{u_n}{u_N}>0,\lim\frac{u_n}{u_N}=0\Rightarrow \lim ln\frac{u_{n+1}}{u_N}=-\infty\Rightarrow\lim S_n\geq\lim\left ( S_{N-1}-c.ln\frac{u_{n+1}}{u_N}\right )=+\infty$ hay $S_n$ phân kỳ, mâu thuẫn với giả sử. Vậy $S_n$ phân kỳ.
 




#741640 Chứng minh F là hình tròn bán kính $\frac{1}{2}...

Gửi bởi poset trong 06-10-2023 - 18:15

Lấy điểm $A$ bất kì trên đường tròn. Kẻ tiếp tuyến $AB,AC$ của $F$ và tiếp tuyến $BD,CE$ của $F$ ($B,C,D,E$ thuộc đường tròn). Giả sử $ABC$ không phải tam giác đều thì ta có $BD//CE,BD\not\equiv CE$, do đó hình $F$ nằm giữa phần tạo bởi dây cung $BD,CE$ và đường tròn (vì $F$ lồi). Tuy nhiên khi đó $AB,AC$ không thể là tiếp tuyến của $F$ (vì $AB,AC$ nằm ngoài phần trên), vô lý.
Vậy $F$ nhận tất cả các cạnh của tất cả các tam giác đều nội tiếp $(O)$ làm tiếp tuyến, hay cụ thể là tất cả các dây cung có khoảng cách tới $O$ là $\frac{1}{2}$. 
Giả sử tồn tại $X$ trên biên của $F$ sao cho $OX\neq \frac{1}{2}$.
Nếu $OX>\frac{1}{2}$, lấy $X'$ trên đoạn $OX$ sao cho $OX'=\frac{1}{2}$, kẻ dây cung $AB$ qua $X'$ vuông góc với $OX'$. Vì $AB$ là tiếp tuyến của $F$ nên $F$ nằm phần của đường tròn chia bời $AB$ chứa $X'$, nhưng khi đó chọn $C$ trên $(O)$ sao cho $ABC$ là tam giác đều thì $CA,CB$ không thể làm tiếp tuyến của $F$, vô lý. Do vậy với mọi $X$ trên biên của $F$ ta có $OX\leq \frac{1}{2}$.
Nếu $OX<\frac{1}{2}$, lấy $X'$ trên tia $OX$ sao cho $OX'=\frac{1}{2}$, kẻ dây cung $AB$ qua $X'$ vuông với $OX'$. Ta có với mọi $Y$ thuộc $AB$, $OY>\frac{1}{2}$ trừ khi $Y\equiv X'$, từ đó ta có $AB$ không cắt $F$, vô lý.
Vậy với mọi điểm $X$ trên biên của $F$, $OX=\frac{1}{2}$, do đó $F$ là đường tròn tâm $O$ bán kính $\frac{1}{2}$ (đpcm)
Có thể thay góc $60^{\circ}$ bằng góc $\alpha \pi ,\alpha \in (0,1)$ với $\alpha$ vô tỉ hoặc có dạng $\frac{m}{n}$ với $m,n$ nguyên dương, $n$ lẻ. Khi đó ta xét các dây cung $A_0A_1,A_1A_2,...,A_{n-1}A_n$ là tiếp tuyến của $F$ (chọn $n$ lẻ sao cho $A_0A_1,A_{n-1}A_n$ không cắt nhau) và làm như trên ta có $F$ là đường tròn tâm $O$ bán kính $cos\frac{\alpha \pi }{2}$.
Với $\alpha$ không có dạng trên, bài toán sẽ phức tạp hơn nhiều và có thể tồn tại $F$ không phải hình tròn thỏa mãn.

https://projecteucli...-01723-6.short 
Với $\alpha$ không có dạng trên thì tồn tại $F$ không phải hình tròn thỏa mãn đề bài. Trường hợp $\alpha=\frac{\pi}{2}$: https://en.wikipedia...Director_circle




#741565 Cho tập $X=\{1;2;3;\ldots ;3000\}$. Có tồn...

Gửi bởi poset trong 29-09-2023 - 22:21

Phân hoạch tập hợp $X$ thành các tập con $X_{2i-1}$ như sau

$$X_{2i-1}=\Big\{2^j(2i-1)\mid j\in\mathbb{N},\ 2^j(2i-1)\le 3000\Big\},\quad i\in\{1,2,\dots,1500\}.$$

Từ đây ta thấy rằng để tập $A$ thỏa đề có số phần tử lớn nhất thì ở mỗi tập hợp $X_{2i-1}$ ta chọn $\left \lfloor \frac{|X_{2i-1}|+1}{2} \right \rfloor$ phần tử (không đồng thời chọn cả $x$ lẫn $2x$). Như vậy số phần tử lớn nhất của $A$ là

$$\sum_{i=1}^{1500}\left \lfloor \frac{|X_{2i-1}|+1}{2} \right \rfloor=\sum_{i=1}^{1500}\left \lfloor \frac{1}{2}\left \lfloor \log_2\left ( \frac{3000}{2i-1} \right )+1 \right \rfloor+\frac{1}{2} \right \rfloor.$$

Một cách để chọn tập $A$ như vậy là lấy tất cả các phần tử có dạng $(2i-1)4^k$. Có thể kiểm tra được rằng tập $A$ thỏa mãn $x\in A\Rightarrow 2x\notin A$ và với cách chọn như vậy số phần tử của $A$ trong mỗi tập hợp $X_{2i-1}$ là lớn nhất ($\left\lfloor\frac{|X_{2i-1}|+1}{2}\right\rfloor$ phần tử) nên $|A|$ cũng lớn nhất. 
Với $k=0$ thì $2i-1\leq 3000\Rightarrow i\leq 1500$ nên có $1500$ cách chọn $i$.
Với $k=1$ thì $4(2i-1)\leq 3000\Rightarrow 2i-1\leq 750\Rightarrow i\leq 375$ nên có $375$ cách chọn $i$.
Với $k=2$ thì $16(2i-1)\leq 3000\Rightarrow 2i-1\leq 187\Rightarrow i\leq 94$ nên có $94$ cách chọn $i$.
...
Cứ như vậy tính được $|A|=1500+375+94+23+6+1=1999<2000$ (gần được), mà ta đã chọn $A$ sao cho $|A|$ lớn nhất nên không tồn tại tập $A$ với $|A|=2000$ như đề bài.
Câu hỏi nhỏ là với $n$ nào thì với tập $X=\{1,2,...,3n\}$ có chứa tập $|A|=2n$ sao cho $x\in A\Rightarrow 2x\notin A$? 




#741430 Ước lượng xác xuất người ra bài trước thắng trong trò Tiến Lên

Gửi bởi poset trong 17-09-2023 - 17:49

Trong trò Tiến lên với $2$ người chơi, ước lượng xác xuất người ra bài trước thắng với các giả định:
- Bài được phát cho mỗi người là hoàn toàn ngẫu nhiên.
- Luật chơi miền Nam tiêu chuẩn với một số thay đổi: không có tới trắng và người ra bài trước được chọn ngẫu nhiên.
- Hai người đều chơi một cách hoàn hảo và họ không có khả năng thu thập thông tin bằng cách nào khác (nhìn cảm xúc, gian lận,...) ngoài phân tích những lá bài đã được đánh.




#740569 Tổng các chữ số trong dãy từ 1 tới n

Gửi bởi poset trong 15-07-2023 - 12:34

Bổ đề
 $\sum_{i=10^km}^{10^k(m+1)-1}s(k)=10^ks(m)+45\cdot 10^{k-1}k$.

Chứng minh
Các số từ $10^km$ đến $10^k(m+1)-1$ có dạng $\overline{ma_1a_2...a_k}, 0\leq a_i\leq 9$. Số hạng đầu tiên từ việc chuỗi $m$ ở đầu xuất hiện $10^k$ lần. Số hạng thứ hai từ việc với $0\leq i\leq 9$, $i$ xuất hiện $10^{k-1}k$ lần trong tất cả chuỗi $k$ số $\overline{a_1a_2...a_k}, 0\leq a_i\leq 9$ ($10^{k-1}$ lần mỗi vị trí $a_j$, có $k$ vị trí như vậy) và $45=0+1+2+...+9$.

Giả sử $n=\overline{a_1a_2...a_m}$. Theo Theorem, ta có: $$\begin{align*} S_n & =\sum_{i=1}^{n}s(i)=\sum_{k=1}^{m}\sum_{i=10^{m-k}\cdot \overline{a_1...a_{k-1}0}}^{10^{m-k}\cdot \overline{a_1...a_{k-1}a_k}-1}s(i) \\ & =\sum_{k=1}^{m}\sum_{i=0}^{a_k-1}\sum_{j=10^{m-k}\cdot\overline{a_1...a_{k-1}i}}^{10^{m-k}\cdot(\overline{a_1...a_{k-1}i}+1)-1}s(j) \\ & =\sum_{k=1}^{m}\sum_{i=0}^{a_k-1}(10^{m-k}(\sum_{j=1}^{k-1}a_j+i)+45\cdot 10^{m-k-1}(m-k)) \\ & =\sum_{k=1}^{m}(a_k(10^{m-k}\sum_{j=1}^{k-1}a_j+45\cdot 10^{m-k-1}(m-k))+10^{m-k}\sum_{i=0}^{a_k-1}i) \\ & =\sum_{1\leq k<l\leq m}10^{m-l}a_ka_l+\sum_{k=1}^{m}(45\cdot 10^{m-k-1}(m-k)a_k+10^{m-k}\frac{a_k(a_k-1)}{2}) \end{align*}$$.




#738991 cho A là tập hợp các số tự nhiên từ 1 đến 200010000, tập A mất đi 99,99% các...

Gửi bởi poset trong 02-05-2023 - 20:10

Cho A là tập hợp các số tự nhiên từ 1 đến 200010000, tập A mất đi 99,99% các số bất kì, chứng minh rằng trong những số còn lại luôn có 3 số lập thành cấp số cộng

Không biết bạn kiếm đâu ra bài này. Đầu tiên là $3$ số chắc khác nhau chứ không thì cái này hiển nhiên. Cái này liên quan định lý Roth về cấp số cộng, nôm na là một tập $S$ gồm một vài số nguyên dương không lớn hơn số $N$ nào đó không chứa $3$ số khác nhau lập thành cấp số cộng sẽ "rất nhỏ", cụ thể $lim_{N\to\infty}sup\frac{|S|}{N}=0$. Để chứng minh người ta thường sử dụng giải tích Forier hay những công cụ cao cấp khác hoặc sơ cấp nhưng rất lằng nhằng và khả năng cao không áp dụng được vào bài này. Đưa số liệu lớn kiểu này chả khác nào bảo mọi người hãy chứng minh định lý Roth đi.

Và hơn nữa, dùng cái trick quen thuộc này thì sẽ thấy đề bài sai. Đề bài có thể hiểu là với mỗi $20001$ số nguyên dương không lớn hơn $200010000$, luôn có $3$ số khác nhau lập thành cấp số cộng. Xét tổng cộng $2^{16}-1=65535>20001$ số gồm các số khi biểu diễn trong cơ số $3$ thì chỉ chứa số $0$ hoặc $1$ và không quá $17$ số. Các số này không vượt quá $3^{16}+3^{15}+...+3^2+3+1=64570081<200010000$ và dễ thấy không tồn tại $3$ số khác nhau lập thành cấp số cộng (để đễ hình dung hãy thử với cơ số $10$). Vậy đề sai.




#738693 Chứng minh rằng dãy $(\lfloor x\rfloor)_{n\in\m...

Gửi bởi poset trong 19-04-2023 - 15:42

Cho $q$ là một số hữu tỷ không nguyên lớn hơn $1$. Xét dãy: $x_1=q, x_{n+1}=q\{x_n\}$. Chứng minh rằng dãy $(\lfloor x\rfloor)_{n\in\mathbb{N}}$ sẽ không bao giờ tuần hoàn, tức không tồn tại $N,k\neq 0$ sao cho $\lfloor x_{n+k}\rfloor=\lfloor x_n\rfloor,\forall n>N$.




#738589 Chứng minh rằng tồn tại số nguyên dương $P$ squarefree sao cho...

Gửi bởi poset trong 13-04-2023 - 23:04

Với đa thức nguyên $Q(X)=b_nX^n+b_{n-1}X^{n-1}+...+b_1X+b_0$, ta định nghĩa $Q(X)[0]=0,Q(X)[x_i]=b_nx_{i+n}+b_{n-1}x_{i+n-1}+...+b_1x_{i+1}+b_0x_i$ và $Q(X)[d_{i_1}x_{i_1}+d_{i_2}x_{i_2}+...+d_{i_m}x_{i_m}]=d_{i_1}Q(X)[x_{i_1}]+d_{i_2}Q(X)[x_{i_2}]+...+d_{i_m}Q(X)[x_{i_m}]$, dễ thấy định nghĩa này hoàn toàn xác định.

Bổ đề
Với hai đa thức nguyên $Q(X),R(X)$, ta có:
a) $(Q(X)+R(X))[d_{i_1}x_{i_1}+d_{i_2}x_{i_2}+...+d_{i_m}x_{i_m}]=Q(X)[d_{i_1}x_{i_1}+d_{i_2}x_{i_2}+...+d_{i_m}x_{i_m}]+R(X)[d_{i_1}x_{i_1}+d_{i_2}x_{i_2}+...+d_{i_m}x_{i_m}]$.
b) $Q(X)[R(X)[d_{i_1}x_{i_1}+d_{i_2}x_{i_2}+...+d_{i_m}x_{i_m}]]=(Q(X)R(X))[d_{i_1}x_{i_1}+d_{i_2}x_{i_2}+...+d_{i_m}x_{i_m}]$.

Chứng minh
 
a) Vì tính chất cộng tính ở định nghĩa, ta chỉ cần chứng minh $(Q(X)+R(X))[x_i]=Q(X)[x_i]+R(X)[x_i],\forall i\in\mathbb{Z}$. Giả sử $Q(X)=b_nX^n+b_{n-1}X^{n-1}+...+b_1X+b_0,R(X)=c_nX^n+c_{n-1}X^{n-1}+...+c_1X+c_0$ với $n=\max(deg(P),deg(Q))$, ta có:
$\begin{matrix}
(Q(X)+R(X))[x_i]=((b_n+c_n)X^n+(b_{n-1}+c_{n-1})X^{n-1}+...+(b_1+c_1)X+b_0+c_0)[x_i]\\
=(b_n+c_n)x_{i+n}+(b_{n-1}+c_{n-1}x_{i+n-1}+...+(b_1+c_1)x_{i+1}+(b_0+c_0)x_i\\
=(b_nx_{i+n}+b_{n-1}x_{i+n-1}+...+b_1x_{i+1}+b_0x_i)+(c_nx_{i+n}+c_{n-1}x_{i+n-1}+...+c_1x_{i+1}+c_0x_i)=Q(X)[x_i]+R(X)[x_i]
\end{matrix}$.
b) Vì tính chất cộng tính ở định nghĩa và a), ta chỉ cần chứng minh $X^n[X^m[x_i]]=X^{n+m}[x_i],\forall i\in\mathbb{Z}$, điều này hiển nhiên.

Bổ đề
Với đa thức nguyên $Q(X)$ và số nguyên tố $p$, ta có $Q^p(X)\equiv Q(X^p)(mod\; p)$.

Chứng minh
Ta có kết quả quen thuộc: $C_p^k\equiv 0(mod\; p)$ với $p$ nguyên tố và $1\leq k\leq p-1$, do đó $(X+Y)^p\equiv X^p+Y^p(mod\; p)$.
Ta quy nạp theo bậc của $Q(X)$, với $deg(Q)=0$ ta có $b^p\equiv b(mod\; p)$, đúng theo định lý Fermat nhỏ.
Giả sử bổ đề đúng với mọi đa thức có bậc nhỏ hơn $n\geq 1$, với $deg(Q)=n$ ta phân tích $Q(X)=bX^n+R(X)$ với $deg(R)<deg(Q)$. Ta có:
$Q^p(X)=(bX^n+R(X))^p\equiv b^pX^{np}+R^p(X)\equiv b(X^p)^n+R(X^p)=Q(X^p)(mod\; p)$
theo giả thuyết quy nạp với đa thức $R(X)$ và định lý Fermat nhỏ. 
Vậy bổ đề đã được chứng minh.

Quay trở lại bài toán. Bằng cách cộng thêm vào các số $k_i$ cùng một số đủ lớn, bài toán vẫn không thay đổi và ta có thể giả sử $k_i$ không âm. Ta đặt $Q(X)=a_1x^{k_1}+a_2x^{k_2}+...+a_nx^{k_n}$ là một đa thức nguyên, theo giả thuyết đề bài ta có $Q(X)[x_i]=0,\forall i\in\mathbb{Z}$. Ta đặt $N=\max_{1\leq i\leq n}(a_i)$. Ta sẽ chứng minh với mọi số nguyên dương $D$ có mọi ước nguyên tố lớn hơn $MNn$, ta có $\sum_{j=1}^na_jx_{i+Dk_j}=Q(X^D)[x_i]=0,\forall i\in\mathbb{Z}$. Ta sẽ chứng minh bài toán mạnh hơn, với mọi đa thức nguyên $R(X)$ có hệ số bị chặn bởi $N$ và số hệ số khác $0$ không quá $n$ thỏa mãn $R(X)[x_i]=0,\forall i\in\mathbb{Z}$, ta có $R(X^D)[x_i]=0,\forall i\in\mathbb{Z}$.
Với $D=p_1^{q_1}p_2^{q_2}...p_m^{q_m},p_i>MNn$, định nghĩa $deg(D)=\sum_{i=1}^mq_i$. Ta sẽ chứng minh bài toán bằng quy nạp theo $deg(D)$:

-Với $deg(D)=0$, tức $D=1$ thì $R(X^D)[x_i]=R(X)[x_i]=0,\forall i\in\mathbb{Z}$ đúng.

-Với $deg(D)=1$, tức $D=p$ với $p>MNn$ là số nguyên tố. Theo Theorem, tồn tại đa thức nguyên $S(X)$ thỏa mãn $R^p(X)=R(X^p)+pS(X)$. Theo Theorem và giả thuyết bài toán ta có:
$0=R^{p-1}(X)[0]=R^{p-1}[R(X)[x_i]]=R^p(X)[x_i]=(R(X^p)+pS(X))[x_i]=R(X^p)[x_i]+pS(X)[x_i],\forall i\in\mathbb{Z}$
tức $R(X^p)[x_i]\equiv 0(mod\; p),\forall i\in\mathbb{Z}$. Ta lại có:
$\left | R(X^p)[x_i] \right |=\left | \sum_{j=1}^{n}b_jx_{i+pl_j} \right |\leq\sum_{j=1}^{n}|b_jx_{i+pl_j}|\leq MNn<p,\forall i\in\mathbb{Z}$ theo giả thuyết của đề bài và đa thức $R(X)$, do đó $R(X^p)[x_i]=0,\forall i\in\mathbb{Z}$, đúng.

-Với $deg(D)>1$, giả sử bài toán đúng với $E$ có mọi ước nguyên tố lớn hơn $MNn$ và $deg(E)=deg(D)-1$. Chọn $p|D,p>MNn$ và $D=pE$ ta có $deg(E)=deg(D)-1$ và $E$ có mọi ước nguyên tố lớn hơn $MNn$, theo giả thuyết quy nạp với $E$ ta có $R(X^E)[x_i]=0,\forall i\in\mathbb{Z}$, ta có $R(X^E)$ có hệ số bị chặn bởi $N$ và số hệ số khác $0$ không quá $n$, do vậy áp dụng bài toán trong trường hợp $p>MNn$ đã chứng minh ở trên ta có $R(X^D)[x_i]=R((X^p)^E)[x_i]=0,\forall i\in\mathbb{Z}$.
Vậy bài toán được chứng minh.

Với $D$ âm và có mọi ước nguyên tố lớn hơn $MNn$, ta đổi dấu $D,k_1,k_2,...,k_n$ và làm tương tự ta cũng có $\sum_{j=1}^na_jx_{i+Dk_j}=Q(X^D)[x_i]=0,\forall i\in\mathbb{Z}$.
Ta chọn $P$ là tích của tất cả số nguyên tố không vượt quá $\max(MNn,2)$ thì $P$ squarefree và nếu $D\equiv 1(mod\; P)$ thì $D$ có mọi ước nguyên tố lớn hơn $MNn$, do đó $\sum_{j=1}^na_jx_{i+Dk_j}=0,\forall i\in\mathbb{Z}$. Vậy $P$ thỏa mãn đề bài.
 




#738085 Công trình nghiên cứu toán học

Gửi bởi poset trong 27-03-2023 - 00:54

Công trình nghiên cứu toán học của mình về lý thuyết số và hình học.

Đặc biệt là về chuỗi hội tụ và phân kỳ, số nguyên tố, Phân số Ai Cập,...

Trên trang web của mình có chứng minh cho phỏng đoán Erdos-Strauss "Chứng minh với n>2 thì 4/n có thể viết dưới dạng tổng 3 phân số Ai Cập khác nhau" đầy đủ trường hợp bằng cách dùng quy luật, thuật toán, công thức, ...

Mình cũng khám phá ra nhiều số mới trong chủ đề về số nguyên tố trong lý thuyết số, đặc biệt là số nguyên tố ngầu, Viprime,... 

MÌnh cũng tìm ra một công thức để kiếm tra số đó có phải là số nguyên tố không và dạng nâng cấp hơn của Fermat, chứng minh phỏng đoán Sierpinski tương tự như phỏng đoán Erdos-Strauss và nhiều hơn nữa.

Mọi người có thể xem trang web của mình để xem công trình nghiên cứu của mình.

Link trang web của mình: https://sites.google...chcom/trang-chủ

Cảm ơn mọi người đã xem.

Nhận xét sơ bộ:

-Không ai lại đi trích dẫn danh ngôn dưới bài nghiên cứu làm gì cả, nó chả giúp mọi người công nhận bạn hay làm bài nghiên cứu hay hơn, thậm chí phản tác dụng. Trừ khi trích dẫn thực sự có liên quan tới nghiên cứu và tác giả phải tinh tế hay có khiếu hài hước. Chứng minh với mọi người bằng tính chính xác và hữu ích ấy, không cần bất kỳ điều gì để nói về bản thân (ngoài địa chỉ liên lạc, lại không thấy đâu) hay "dạy" ai đó biết công nhận người khác.

-Nhiều thứ mình không biết nó là định lý, bổ đề, vấn đề hay giả thiết.

-Bài tiếng Anh và có trích dẫn ảnh tiếng Việt về định lý Beal??!! Viết bằng tiếng Việt có khi hay hơn.

-Nghiên cứu Toán Học, không phải nơi của những công thức lằng nhằng J4F (khiếu hài hước tệ quá) và không phải nơi giới thiệu từ vựng tiếng Anh mới?!

-Các hình rất dễ nhìn và dễ hiểu, đặc biệt là hình chồng $16$ hình tròn lên nhau thì phải, thông cảm mình đếm hơi tệ.

-Phần chuyên môn căn bản thì nó mang lại cho người đọc rất nhiều hy vọng: hy vọng thuật toán này hoạt động, hy vọng mệnh đề này đúng, hy vọng bạn làm bài nghiên cứu đẹp hơn xíu,...

Không biết có đủ rảnh để nhận xét chi tiết không...




#737804 Nếu bạn dùng Gmail, xin hãy giúp diễn đàn một việc nhỏ

Gửi bởi poset trong 16-03-2023 - 20:52

:lol:




#737768 Giới hạn nhận thức của con người

Gửi bởi poset trong 15-03-2023 - 21:18

Để có thể tính toán, ta sẽ đặt hai giả định sau:

-Sai số tương đối là như nhau với hai hình đồng dạng, ví dụ nếu sai số khi thấy ở một hình là $1cm$ thì sai số khi thấy ở hình to gấp đôi là $2cm$ (luật Weber-Fechner, en.wikipedia.org/wiki/Weber–Fechner_law) và cũng không thay đổi khi dùng các phép dời hình (*).
-Nếu có một thuật toán giúp xử lý vấn đề gì đó, thì sai số của thuật toán đó không nhỏ hơn khi ước lượng bình thường. Cái này là giả định riêng của mình, cơ bản là coi hệ thống xử lý thị giác của con người khá "hoàn hảo", khi không có cách tư duy nào làm tăng độ chính xác của nó (**).
Đầu tiên, ta xét vấn đề đơn giản sau: Cho đoạn thẳng $AB=1$ và số thực $x\in [0,1]$, tìm điểm $C$ nằm trên $AB$ sao cho $AC=x$ (đặt vấn đề thế này để tránh thiên kiến nhận thức), $f^+(x),f^-(x)$ lần lượt là cận trên và cận dưới của $\frac{AC}{AB}$ với $C$ ước lượng bằng mắt thường. Ta sẽ tìm hiểu các tính chất của $f^+(x),f^-(x)$:

$f^-(0)=f^+(0)=0,f^-(1)=f^+(1)=1$, rõ ràng chọn $C$ trùng với $A$ hoặc $B$ là được.
$0\leq f^-(x)\leq x\leq f^+(x)\leq 1$, rõ ràng ta sẽ chọn $C$ nằm trên $AB$.

$f^+(x)+f^-(1-x)=1$ do tính đối xứng.

Xét $x,y\in [0,1]$. Ta tìm sẽ điểm $C,D$ nằm trên $AB$ sao cho $AC=x,AD=xy$ bằng mắt thường. Xét hai cách:

-Ta tìm điểm $C$ trước rồi tìm điểm $D$ trên $AC$ sau. Điểm $C$ được chọn sẽ nằm trên khoảng $[f^-(x),f^+(x)]$, vì giả định (*) nên điểm $D$ được chọn sẽ nằm trên khoảng $[f^-(x),f^-(y),f^+(x)f^+(y)]$. Vì giả định (**) ta có bất phương trình hàm $f^-(x)f^-(y)\leq f^-(xy) ,f^+(x)f^+(y)\geq f^+(xy),\forall x,y\in [0,1]$.

-Ta tìm điểm $D$ trước rồi tìm điểm $C$ sao cho điểm $D'$ thỏa mãn $AD'=yAC$ trùng với $D$. Điểm $D$ được chọn sẽ nằm trên khoảng $[f^-(xy),f^+(xy)]$, điểm $C$ sẽ thỏa mãn $D\in [f^-(y)AC,f^+(y)AC]$, tức điểm $C$ được chọn sẽ nằm trên khoảng $[\frac{f^-(xy)}{f^+(y)},\frac{f^+(xy)}{f^-(y)}]$. Vì giả định (**) ta có bất phương trình hàm $\frac{f^-(xy)}{f^+(y)}\leq f^-(x)\Rightarrow f^-(xy)\leq f^-(x)f^+(y),\frac{f^+(xy)}{f^-(y)}\geq f^+(x)\Rightarrow f^+(xy)\geq f^+(x)f^-(y),\forall x,y\in [0,1]$.
(đang giải quyết cái bất phương trình hàm của nợ này :wacko: )
 

Mới nhận ra góc nhìn/point of view có thể là dữ kiện còn thiếu. Ta giả định khoảng sai số sẽ giữ nguyên dưới mọi góc nhìn, căn bản là là phép biến đổi xạ ảnh, tức làm mạnh hơn giả định thứ nhất bằng thay phép đồng dạng bằng phép biến đổi xạ ảnh. Lấy điểm $C_-,C_+$ sao cho $AC_-=f^-(x),AC_+=f^+(x)$. Đặt $\alpha=f^-(\frac{1}{2})=1-f^-(\frac{1}{2})\leq\frac{1}{2}$. Ta đổi "góc nhìn" sao cho "thấy" $C$ là trung điểm $AB$, đưa bài toán về $x=\frac{1}{2}$. Qua phép biến đổi xạ ảnh không làm thay đổi tỷ số kép, tức $(A,B,C,C_-)=(A',B',C',C'_-),(A,B,C,C_+)=(A',B',C',C'_+)\Rightarrow\frac{x(1-f^-(x))}{(1-x)f^-(x)}=\frac{1-\alpha}{\alpha},\frac{x(1-f^+(x))}{(1-x)f^+(x)}=\frac{\alpha}{1-\alpha}$, giải ra ta được $f^-(x)=\frac{\alpha x}{2\alpha x-x-\alpha+1},f^+(x)=\frac{(1-\alpha)x}{(1-2\alpha)x+\alpha}$. Check thử thì tất cả các điều kiện còn lại đều thỏa.
.




#737725 Thông báo về việc bảo trì diễn đàn

Gửi bởi poset trong 14-03-2023 - 15:51

Không biết có tính năng tìm kiếm những bài chưa có câu trả lời không nhỉ? Chứ có thể có những bài hay trước kia nhưng không ai biết tới vì khó tìm.